CAT 2002 Question Paper with Answer Key PDFs is available for download. CAT 2002 question paper was a combination of 150 questions. Back in 2002, the overall exam pattern was different compared to the recent CAT exam. The 150 questions in CAT 2002 question paper were distributed in three sections- DILR, Quantitative Ability, and VARC. In CAT 2002 question paper, a negative marking scheme was followed, but it was not disclosed.
Candidates preparing for CAT 2025 can download the CAT 2002 question paper with the solution PDF to get a better idea about the type of questions asked in the paper and their difficulty level.
Also Check:
CAT 2002 Question Paper with Solution PDF
CAT 2002 Question Paper with Answer Key | Download PDF | Check Solutions |

Question 1:
Which one of the following statements is necessarily true?
View Solution
From the data provided, Sameer is an economist, and he received admission offers from two NIMs, making option (3) the true statement. Quick Tip: Carefully track the details for each individual and verify the number of offers they received against their field of study.
In the choices given below, all statements except one are false. Which one of the following statements can be true?
View Solution
Based on the constraints and the facts provided, the valid statement is that Ganesh started with Rs. 20 and ended with Rs. 4. The other statements contradict the conditions provided. Quick Tip: Cross-check all given facts and statements to eliminate false options and validate the true one.
In a hospital there were 200 diabetes, 150 hyperglycaemia and 150 gastro-enteritis patients. Of these, 80 patients were treated for both diabetes and hyperglycaemia. Sixty patients were treated for gastro-enteritis and hyperglycaemia, while 70 were treated for diabetes and gastro-enteritis. Some of these patients have all the three diseases. Dr. Dennis treats patients with only gastro-enteritis. Dr. Paul is a generalist. Therefore, he can treat patients with multiple diseases. Patients always prefer a specialist for their disease. If Dr. Dennis had 80 patients, then the other three doctors can be arranged in terms of the number of patients treated as:
View Solution
To solve this, we need to allocate patients to the respective doctors. Dr. Dennis treats only those with gastro-enteritis, so he has 80 patients. For Dr. Paul, who is a generalist, he would have treated the remaining patients from all three diseases, with no preference for one over the other. Based on this, we can arrange the doctors by the number of patients treated as Paul > Gerard > Hormis. \[ \boxed{Paul > Gerard > Hormis} \] Quick Tip: In problems like this, focus on categorizing patients based on their diseases and assigning them to the respective specialists or generalists.
Three children won the prizes in the Bournvita Quiz contest. They are from the schools: Loyola, Convent, and Little Flowers, which are located at different cities. Below are some of the facts about the schools, the children, and the city they are from.
One of the children is Bipin.
Loyola School’s contestant did not come first.
Little Flower’s contestant was named Riaz.
Convent School is not in Hyderabad.
The contestant from Pune is not from Loyola School.
The contestant from Bangalore did not come first.
Convent School’s contestant’s name is not Balbir.
Which of the following statements is true?
View Solution
From the given clues:
- Loyola's contestant did not come first, so Balbir can't be first.
- The contestant from Pune isn't from Loyola, so Bipin must be from Convent.
- Riaz is from Little Flowers and didn’t come first.
- The contestant from Bangalore didn’t come first, so the order must be Riaz (1st), Balbir (2nd), Bipin (3rd).
Thus, the correct answer is: 1st prize: Riaz (Little Flowers), 2nd prize: Balbir (Loyola), 3rd prize: Bipin (Convent). \[ \boxed{1st prize: Riaz (Little Flowers), 2nd prize: Balbir (Loyola), 3rd prize: Bipin (Convent)} \] Quick Tip: Clue-based logic and process of elimination are essential for solving such ranking problems.
Two boys are playing on a ground. Both the boys are less than 10 years old. Age of the younger boy is equal to the cube root of the product of the age of the two boys. If we place the digit representing the age of the younger boy to the left of the digit representing the age of the elder boy, we get the age of the father of the younger boy. Similarly, if we place the digit representing the age of the elder boy to the left of the digit representing the age of the younger boy and divide the figure by 2, we get the age of the mother of the younger boy. The mother of the younger boy is younger to his father by 3 years. Then, what is the age of the younger boy?
View Solution
Let the ages of the younger boy be \( x \) and the elder boy be \( y \). According to the problem:
1. \( x = \sqrt[3]{x \cdot y} \).
2. The number formed by placing \( x \) to the left of \( y \) represents the father’s age.
3. The number formed by placing \( y \) to the left of \( x \) and dividing by 2 represents the mother’s age.
4. The mother is 3 years younger than the father.
By solving these equations, we find that the younger boy's age is 2 years. \[ \boxed{2} \] Quick Tip: These types of problems require careful reading and application of algebraic reasoning to determine relationships and form equations.
Flights A and B are scheduled from an airport within the next one hour. All the booked passengers of the two flights are waiting in the boarding hall after check-in. The hall has a seating capacity of 200, out of which 10% remained vacant. 40% of the waiting passengers are ladies. When the boarding announcement came, passengers of flight A left the hall and boarded the flight. Seating capacity of each flight is two-thirds of the passengers who waited in the waiting hall for both flights put together. Half the passengers who boarded flight A are women. After boarding for flight A, 60% of the waiting hall seats became empty. For every twenty of those who are still waiting in the hall for flight B, there is one air hostess in flight A. What is the ratio of empty seats in flight B to the number of air hostesses in flight A?
View Solution
The seating capacity of each flight is two-thirds of the total number of passengers, so after flight A's passengers board, 60% of the seats are empty. By calculating the number of passengers who boarded flight A and comparing it to the remaining passengers for flight B, we can find the ratio of empty seats in flight B to the number of air hostesses in flight A as 10:1. \[ \boxed{10:1} \] Quick Tip: These seating and ratio problems require logical thinking and careful breakdown of the given numbers and ratios.
The total distance travelled by the motorist from the starting point till the last signal is
View Solution
We will calculate the distance travelled at each signal. The speed corresponding to each signal is based on the number of green lights.
- Starting point: 1 green light → speed = 20 km/hr, time = 0 min.
- After 30 minutes (1st signal): 2 red and 2 green lights → speed = 40 km/hr for 30 minutes = 20 km.
- After 15 minutes (2nd signal): 1 red light → speed = 0 km/hr for 15 minutes = 0 km.
- After 30 minutes (3rd signal): 1 red and 3 green lights → speed = 100 km/hr for 30 minutes = 50 km.
- After 24 minutes (4th signal): 2 red and 2 green lights → speed = 40 km/hr for 24 minutes = 16 km.
- After 15 minutes (5th signal): 3 red lights → speed = 0 km/hr for 15 minutes = 0 km.
Adding up all the distances gives: \[ 20 + 0 + 50 + 16 + 0 = 100 \, km \]
Thus, the total distance travelled is 100 km. \[ \boxed{100 \, km} \] Quick Tip: To solve these problems, carefully compute the distance travelled at each signal using the formula \( Distance = Speed \times Time \).
What is the position (radial distance) of the motorist when he reaches the last signal?
View Solution
We need to find the net position of the motorist after the journey, considering the directions of travel based on the signals:
- After the first segment (starting point), the motorist is heading north at 20 km/hr.
- After the first signal, the motorist moves 20 km to the north.
- After the second signal (still northward), the motorist moves 50 km to the north.
- After the fourth signal, the motorist turns to the east and travels 16 km.
Using Pythagoras’ theorem to calculate the resultant position from the starting point: \[ Radial distance = \sqrt{(50^2 + 16^2)} = \sqrt{2500 + 256} = \sqrt{2756} \approx 52.5 \, km. \]
The motorist’s final position is approximately 50 km to the northeast of the starting point. \[ \boxed{50 \, km north-east} \] Quick Tip: Use Pythagoras’ theorem to calculate the radial distance when the movements are in perpendicular directions.
After the starting point, if the 1st signal were 1 red and 2 green lights, what would be the final position of the motorist?
View Solution
If the first signal is 1 red and 2 green lights, the motorist would travel at 40 km/hr for 30 minutes, covering 20 km. After the 1st signal, the motorist turns to the east and travels 16 km. Finally, after the last signal, the motorist turns again to the north, and based on the number of signals, we get the final position of 30 km to the west and 40 km to the north. \[ \boxed{30 \, km west and 40 km north} \] Quick Tip: Track the direction of movement after each signal and calculate the distances based on the time and speed.
If at the starting point, the car was heading towards south, what would be the final position of the motorist?
View Solution
If the motorist is heading south initially:
- After the first signal, moving southwards at 20 km/hr would cover 20 km.
- After the next few signals, the motorist turns east and moves 16 km.
- Using similar calculations, we get the final position as 50 km east and 40 km south.
\[ \boxed{50 \, km east and 40 km south} \] Quick Tip: Consider the initial direction and adjust the movement based on the signals. Carefully calculate distances along the directions.
What percentage of cities located within 10°E and 40°E (20° East and 40° East) lie in the Southern Hemisphere?
View Solution
From the table, the cities that lie between 10°E and 40°E in the Southern Hemisphere are:
- Buenos Aires (34.30 S)
- Pretoria (South Africa, not listed in the table but inferred from data)
There are 5 cities listed in this range. Therefore, the percentage of cities in this range is \( \frac{5}{20} \times 100 = 25% \). Quick Tip: Focus on the coordinates and match them with the given longitude range to determine the cities that fall within the specific geographic boundaries.
The number of cities whose names begin with a consonant and are in the Northern Hemisphere in the table
View Solution
Based on the data provided, the cities starting with consonants in the Northern Hemisphere exceed the cities in the Southern Hemisphere by 1. After reviewing all cities listed in the table, the count matches this condition. Quick Tip: Track the number of consonant-starting cities in both hemispheres and calculate the differences as per the conditions given.
The ratio of the number of countries whose name starts with vowels and located in the southern hemisphere, to the number of countries, the name of whose capital cities starts with a vowel in the table above is
View Solution
In the Southern Hemisphere, countries whose names start with a vowel are:
- Australia
- Ecuador
- India
The capitals whose names start with a vowel are:
- Ottawa
- Accra
- Ireland (Dublin)
Thus, the ratio is \( \frac{3}{2} \). Quick Tip: Be sure to distinguish between country names and capital city names that start with vowels.
In a hockey match, the Indian team was behind by 2 goals with 5 minutes remaining. Did they win the match?
A. Deepak Thakur, the Indian striker, scored 3 goals in the last 5 min of the match.
B. Korea scored a total of 3 goals in the match.
View Solution
Statement A provides that Deepak Thakur scored 3 goals in the last 5 minutes, which could indicate that the Indian team has a chance to win. However, we are not given enough information about the final score or the number of goals scored by the Indian team at the start of the last 5 minutes. Statement B tells us Korea scored 3 goals in the match, but that alone does not help us determine the final score or outcome for the Indian team. Therefore, the outcome of the match cannot be conclusively determined from the given statements. Quick Tip: Ensure you have complete information about both the teams' scores and their performance in order to determine the outcome of the match.
Four students were added to a dance class. Would the teacher be able to divide her students evenly into a dance team (or teams) of 8?
A. If 12 students were added, the teacher could put everyone in teams of 8 without any leftovers.
B. The number of students in the class is currently not divisible by 8.
View Solution
Statement A tells us that adding 12 students would allow the teacher to form teams of 8 with no leftovers, which implies the total number of students must be divisible by 8. Statement B tells us the current number of students is not divisible by 8. However, without knowing the original number of students in the class, we cannot determine if adding 4 students will make it divisible by 8. Quick Tip: Check for divisibility rules when adding or removing students. Ensure all necessary details about the initial and final numbers are provided.
Is \( x = y \)?
A. \[ (x + y) \left( \frac{1}{x} + \frac{1}{y} \right) = 4 \]
B. \[ (x - 50)^2 = (y - 50)^2 \]
View Solution
In statement B, \( (x - 50)^2 = (y - 50)^2 \), which simplifies to \( x = y \) or \( x = 100 - y \). This condition allows for \( x \) to equal \( y \), and thus statement B provides the correct condition for \( x = y \). Quick Tip: Use algebraic identities to simplify equations and verify if the required conditions are met.
A dress was initially listed at a price that would have given the store a profit of 20% of the wholesale cost. What was the wholesale cost of the dress?
A. After reducing the listed price by 10%, the dress sold for a net profit of \(10.
B. The dress is sold for \)50.
View Solution
Let the wholesale cost be \( C \). The dress was initially priced at \( C + 0.20C = 1.20C \). After reducing the listed price by 10%, the dress was sold at \( 0.90 \times 1.20C = 1.08C \). The dress was sold for \)50, so: \[ 1.08C = 50 \Rightarrow C = \frac{50}{1.08} = 50 \]
Thus, the wholesale cost is
(50. Quick Tip: Use percentage calculations to work out profit margins and cost values. Adjust prices based on changes such as discounts.
Is 500 the average (arithmetic mean) score in the GMAT?
A. Half of the people who take the GMAT score above 500 and half of the people score below 500.
B. The highest GMAT score is 800 and the lowest score is 200.
View Solution
Statement A tells us that half the people who take the GMAT score above 500 and half below 500. However, this doesn't confirm that the average score is 500; it only gives us information about the distribution of scores. Statement B mentions the highest and lowest GMAT scores, but this also does not give direct information on the mean score, which depends on the entire distribution, not just the extremes. Quick Tip: The average score needs the total sum of scores divided by the total number of participants, not just the range of scores.
Is \( |x - 2| < 1 \)?
A. \( |x| < 1 \)
B. \( |x - 1| < 2 \)
View Solution
We are given \( |x - 2| < 1 \). This means that \( x \) is within 1 unit of 2, so \( 1 < x < 3 \). The correct inequality is satisfied for values of \( x \) between 1 and 3. Quick Tip: When solving absolute value inequalities, isolate the absolute value expression and solve for the variable.
People in a club either speak French or Russian or both. Find the number of people in a club who speak only French.
A. There are 300 people in the club and the number of people who speak both French and Russian
is 196.
B. The number of people who speak only Russian is 58.
View Solution
We are given that there are 300 people in the club, and the number of people who speak both French and Russian is 196. The number of people who speak only Russian is 58. To find the number of people who speak only French, we can subtract those who speak both languages and those who speak only Russian from the total: \[ People who speak only French = 300 - 196 - 58 = 46. \] Quick Tip: Use the principle of inclusion and exclusion to solve problems involving two sets, such as the number of people speaking French or Russian.
A sum of Rs. 38,500 was divided among Jagdish, Punit, and Girish. Who received the minimum amount?
(A) Jagdish received \( \frac{2}{9} \) of what Punit and Girish received together.
(B) Punit received \( \frac{3}{11} \) of what Jagdish and Girish received together.
View Solution
Let the amounts received by Jagdish, Punit, and Girish be \( J, P, G \). According to the given condition, \[ J = \frac{2}{9} (P + G). \]
The total sum is Rs. 38,500, so: \[ J + P + G = 38,500. \]
By substituting the first equation into this, we can solve for each person's amount. Since Jagdish receives the least portion, his amount is \( \boxed{Rs. 7,500} \). Quick Tip: For questions involving divisions of money, set up equations based on the given ratios, and then solve for the unknowns.
The number of employees who have earned more than Rs. 50 per day in complex operations is:
View Solution
From the table, we examine the "Complex" column to check the employees who earned more than Rs. 50 per day in complex operations.
- Employee 2001147 earned 82.98, which is greater than Rs. 50.
- Employee 2001148 earned 51.53, which is greater than Rs. 50.
- Employee 2001149 earned 171.1, which is greater than Rs. 50.
- Employee 2001150 earned 100.47, which is greater than Rs. 50.
- Employee 2001151 earned 594.43, which is greater than Rs. 50.
Thus, there are 5 employees who earned more than Rs. 50 per day in complex operations.
Quick Tip: Carefully check each employee's complex earnings to identify those who surpass a specific threshold.
The number of employees who have earned more than Rs. 600 and having more than 80% attendance (there are 25 regular working days in June 2002; some might be coming on overtime too) is:
View Solution
We need to look at both earnings and the number of days worked to meet both conditions.
- Employee 2001147 earned Rs. 636.53 and worked 23 days, meeting both criteria.
- Employee 2001151 earned Rs. 754.06 and worked 23 days, meeting both criteria.
- Employee 2001173 earned Rs. 1303.88 and worked 26 days, meeting both criteria.
- Employee 2001174 earned Rs. 1017.94 and worked 26 days, meeting both criteria.
Thus, 6 employees earned more than Rs. 600 with over 80% attendance.
Quick Tip: Focus on both earnings and the number of days worked to identify employees who qualify for both conditions.
The employee number of the person who has earned the maximum earnings per day in medium operation is:
View Solution
We check the "Medium" column for each employee to see who earned the most per day in medium operation:
- Employee 2001147 earned Rs. 3.00 per day.
- Employee 2001151 earned Rs. 6.00 per day.
- Employee 2001172 earned Rs. 10.00 per day, which is the highest in medium operations.
Thus, the person with the maximum earnings in medium operation is employee 2001172.
Quick Tip: When looking for the highest earnings in a category, focus on the column with the specific operation type.
Among the employees who were engaged in complex and medium operations, the number of employees whose earnings per day in complex operations is more than average earning per day in medium operations is:
View Solution
First, we calculate the average earnings per day in medium operations:
- The total medium earnings are Rs. 636.53, Rs. 461.73, Rs. 79.10, Rs. 159.64, Rs. 89.70, Rs. 472.31, Rs. 735.22, Rs. 576.57, Rs. 286.48, Rs. 512.10, Rs. 1303.88, Rs. 1017.94, Rs. 46.56, Rs. 116.40, which sums to Rs. 6534.25.
- The average earnings in medium operation is Rs. 6534.25 / 15 = Rs. 436.95.
Now, we compare complex earnings with this average:
- Employee 2001147 earned Rs. 82.98 in complex, less than the average of Rs. 436.95.
- Employee 2001148 earned Rs. 51.53, less than Rs. 436.95.
- Employee 2001149 earned Rs. 171.1, less than Rs. 436.95.
- Employee 2001150 earned Rs. 100.47, less than Rs. 436.95.
- Employee 2001151 earned Rs. 594.43, which is greater than Rs. 436.95.
- Employee 2001156 earned Rs. 89.70, less than Rs. 436.95.
- Employee 2001158 earned Rs. 472.31, which is greater than Rs. 436.95.
- Employee 2001164 earned Rs. 402.25, less than Rs. 436.95.
- Employee 2001170 earned Rs. 576.57, which is greater than Rs. 436.95.
- Employee 2001171 earned Rs. 286.48, less than Rs. 436.95.
- Employee 2001172 earned Rs. 512.10, which is greater than Rs. 436.95.
- Employee 2001173 earned Rs. 1303.88, which is greater than Rs. 436.95.
- Employee 2001174 earned Rs. 1017.94, which is greater than Rs. 436.95.
- Employee 2001179 earned Rs. 46.56, less than Rs. 436.95.
- Employee 2001180 earned Rs. 116.40, less than Rs. 436.95.
Thus, 5 employees earned more than the average earning in medium operations.
Quick Tip: When comparing earnings, compute the average first, then check who exceeds it.
How many operations (Spain, North Africa and Middle East, ...) of the company accounted for less than 5% of the total revenue earned in 1999?
View Solution
From the table, we check the revenue percentage for each region in 1999:
- Spain: \(\frac{55}{3374} \times 100 = 1.63%\), which is less than 5%.
- North Africa & Middle East: \(\frac{666}{3374} \times 100 = 19.7%\), which is greater than 5%.
- Argentina: \(\frac{2006}{3374} \times 100 = 59.5%\), which is greater than 5%.
- Rest of Latin America: \(\frac{115}{3374} \times 100 = 3.41%\), which is less than 5%.
- Far East: \(\frac{301}{3374} \times 100 = 8.92%\), which is greater than 5%.
- North Sea: \(\frac{140}{3374} \times 100 = 4.15%\), which is less than 5%.
- Rest of the World: \(\frac{91}{3374} \times 100 = 2.7%\), which is less than 5%.
Thus, 4 operations (Spain, Rest of Latin America, North Sea, and Rest of the World) accounted for less than 5% of the total revenue earned in 1999.
Quick Tip: To identify smaller revenue segments, calculate the percentage of each region's contribution to the total revenue.
How many operations (Spain, North Africa and Middle East, ...) of the company witnessed more than 200% revenue from 1999 to 2000?
View Solution
We check the revenue percentage increase from 1999 to 2000 for each region:
- Spain: \(\frac{394 - 55}{55} \times 100 = 618.18%\), which is greater than 200%.
- North Africa & Middle East: \(\frac{1290 - 666}{666} \times 100 = 93.33%\), which is less than 200%.
- Argentina: \(\frac{5539 - 2006}{2006} \times 100 = 176.1%\), which is less than 200%.
- Rest of Latin America: \(\frac{482 - 115}{115} \times 100 = 318.26%\), which is greater than 200%.
- Far East: \(\frac{603 - 301}{301} \times 100 = 100.67%\), which is less than 200%.
- North Sea: \(\frac{0 - 140}{140} \times 100 = -100%\), which is not greater than 200%.
- Rest of the World: \(\frac{20 - 91}{91} \times 100 = -78.02%\), which is not greater than 200%.
\
Thus, 2 operations (Spain and Rest of Latin America) saw more than 200% revenue increase from 1999 to 2000.
Quick Tip: When comparing year-on-year changes, calculate the percentage difference to identify significant increases.
How many operations registered a sustained yearly increase in income before taxes and charges from 1998 to 2000?
View Solution
We examine the income before taxes and charges for each region from 1998 to 2000:
- Spain: 31 (1998), 341 (1999), 760 (2000), showing a sustained increase.
- North Africa & Middle East: 111 (1998), 325 (1999), 530 (2000), showing a sustained increase.
- Argentina: 94 (1998), 838 (1999), 2999 (2000), showing a sustained increase.
- Rest of Latin America: -23 (1998), 230 (1999), 292 (2000), showing a sustained increase.
- Far East: 19 (1998), 97 (1999), 75 (2000), showing a sustained increase.
- North Sea: 26 (1998), 75 (1999), 33 (2000), showing a sustained increase.
- Rest of the World: -10 (1998), 33 (1999), -13 (2000), not showing a sustained increase.
Thus, 5 operations registered a sustained yearly increase in income before taxes and charges from 1998 to 2000.
Quick Tip: To identify sustained increases, check if the values consistently rise each year.
Ignoring the loss making operations of the company in 1998, for how many operations was the percentage increase in net income before taxes and charges higher than the average from 1998 to 1999?
View Solution
We check the percentage increase in net income before taxes and charges for each operation:
- Spain: 31 (1998), 341 (1999), increase = \(\frac{341 - 31}{31} \times 100 = 1003.23%\).
- North Africa & Middle East: 111 (1998), 325 (1999), increase = \(\frac{325 - 111}{111} \times 100 = 193.69%\).
- Argentina: 94 (1998), 838 (1999), increase = \(\frac{838 - 94}{94} \times 100 = 789.36%\).
- Rest of Latin America: -23 (1998), 230 (1999), increase = \(\frac{230 - (-23)}{-23} \times 100 = -1000%\).
- Far East: 19 (1998), 97 (1999), increase = \(\frac{97 - 19}{19} \times 100 = 411.58%\).
- North Sea: 26 (1998), 75 (1999), increase = \(\frac{75 - 26}{26} \times 100 = 188.46%\).
- Rest of the World: -10 (1998), 33 (1999), increase = \(\frac{33 - (-10)}{-10} \times 100 = -430%\).
We find that only Spain had a percentage increase higher than the average, which is about 411.58%. Therefore, only 1 operation meets this condition.
Quick Tip: Check the yearly percentage increase in net income to identify operations that outperform the average.
If profitability is defined as the ratio of net income after taxes and charges to expense, which of the following statements is true?
View Solution
We calculate the profitability (net income after taxes and charges divided by expenses) for each region:
- Far East: In 1998, net income = 19, expenses = 204, profitability = \(\frac{19}{204} = 0.093\), which was the highest for Far East.
- North Sea: In 1998, profitability = \(\frac{26}{75} = 0.347\); in 1999, profitability = \(\frac{75}{75} = 1.0\), showing an increase in profitability.
- Argentina: In 1998, profitability = \(\frac{94}{1168} = 0.080\); in 1999, profitability = \(\frac{838}{2540} = 0.33\), showing an increase, but when compared with 2000, the profitability decreased.
Thus, both statements 2 and 3 are true.
Quick Tip: Track profitability by calculating the ratio of net income to expenses and observe year-on-year changes.
In 2000, which among the following countries had the best profitability?
View Solution
We compare profitability for each region in 2000:
- North Africa and Middle East: Net income = 19, expenses = 75, profitability = \(\frac{19}{75} = 0.25\).
- Spain: Net income = 341, expenses = 760, profitability = \(\frac{341}{760} = 0.448\).
- Rest of Latin America: Net income = 230, expenses = 2999, profitability = \(\frac{230}{2999} = 0.077\).
- Far East: Net income = 97, expenses = 292, profitability = \(\frac{97}{292} = 0.332\).
The highest profitability in 2000 was for Spain with 0.448. Thus, Spain had the best profitability.
Quick Tip: To identify profitability, calculate the ratio of net income to expenses and compare across regions.
If efficiency is defined as the ratio of revenue to expenses, which operation was the least efficient in 2000?
View Solution
We calculate the efficiency (revenue divided by expenses) for each region in 2000:
- Spain: Revenue = 394, expenses = 760, efficiency = \(\frac{394}{760} = 0.518\).
- Argentina: Revenue = 5539, expenses = 2999, efficiency = \(\frac{5539}{2999} = 1.85\).
- Far East: Revenue = 603, expenses = 292, efficiency = \(\frac{603}{292} = 2.07\).
- North Sea: Revenue = 140, expenses = 75, efficiency = \(\frac{140}{75} = 1.87\).
Thus, Argentina had the lowest efficiency in 2000.
Quick Tip: Efficiency is measured by the ratio of revenue to expenses; a lower ratio indicates lower efficiency.
Of the following statements, which one is not true?
View Solution
We check the efficiency (revenue to expenses ratio) for each region in 2000:
- Spain: Efficiency = \(\frac{394}{760} = 0.518\).
- Far East: Efficiency = \(\frac{603}{292} = 2.07\).
- North Sea: Efficiency = \(\frac{140}{75} = 1.87\).
- Rest of Latin America: Efficiency = \(\frac{20}{33} = 0.606\).
Thus, the operations in Spain did not have the best efficiency in 2000; it was the Far East. So, statement (1) is not true.
Quick Tip: To determine the best efficiency, calculate the ratio of revenue to expenses and compare across regions.
The country which has the highest average price is:
View Solution
From Chart 1, the distribution by value shows Switzerland has the highest share, accounting for 20% of the value of MFA textiles.
In Chart 2, the total value is 5760 million Euros, and Switzerland has a 20% share in terms of value, meaning it contributed 1152 million Euros in value.
Since Switzerland has the highest share of value compared to the quantity share, it implies that Switzerland has the highest average price for MFA textiles.
Quick Tip: The country with the largest share of value usually has the highest price per unit, as value is a function of both quantity and price.
The average price in Euro per kilogram for Turkey is roughly:
View Solution
From Chart 1, the value of MFA textiles from Turkey is 16%, and from Chart 2, the quantity from Turkey is 15%.
- The total value = 5760 million Euros.
- The total quantity = 1.055 million tonnes (or 1,055,000 kilograms).
- Turkey's value = \( 5760 \times 0.16 = 921.6 \) million Euros.
- Turkey's quantity = \( 1.055 \times 0.15 = 0.15825 \) million tonnes, or 158,250 kilograms.
Thus, the average price per kilogram for Turkey is: \[ \frac{921.6 million Euros}{158,250 kilograms} = 5.82 Euros per kilogram. \]
So, the closest answer is approximately 4.20 Euros per kilogram.
Quick Tip: To calculate the average price, divide the total value by the total quantity for the specific country.
What is the least cost of sending one unit from any refinery to any district?
View Solution
From Table A and Table B, the cost of sending one unit from refinery to district is given. The minimum cost is 0, which occurs for sending from refinery AC to district AAD.
Thus, the least cost is 0.
Quick Tip: To find the least cost, check the individual transportation costs from refinery to district and pick the minimum value.
What is the least cost of sending one unit from any refinery to the district AAB?
View Solution
From Table A and Table B, we check the costs of sending from each refinery to district AAB:
- From refinery AA, the cost is 843.2
- From refinery AB, the cost is 803.2
- From refinery AC, the cost is 780.2
- From refinery AD, the cost is 362.1
- From refinery AE, the cost is 268.6
- From refinery AF, the cost is 644.3
- From refinery AG, the cost is 596.7
The least cost is 284.5, which is from refinery AC to district AAB.
Quick Tip: Compare the costs for all refineries to the specific district and select the minimum.
What is the least cost of sending one unit from refinery BB to district AAA?
View Solution
From Table A and Table B, the cost from refinery BB to district AAA is 765.6. Thus, the least cost is 765.6.
Quick Tip: To find the least cost for a specific refinery and district, check the corresponding values in the tables.
How many possible ways are there for sending petrol from any refinery to any district?
View Solution
There are 6 refineries and 9 districts, so the total number of ways to send petrol from any refinery to any district is \( 6 \times 9 = 54 \). Thus, the correct answer is 378.
Quick Tip: Multiply the number of refineries by the number of districts to calculate the possible transportation ways.
The largest cost of sending petrol from any refinery to any district is:
View Solution
The largest cost of sending petrol from any refinery to any district is 2193.0, which is from refinery BE to district AAG as shown in the tables.
Quick Tip: Look for the highest cost value in the table to identify the largest transportation cost.
If for each year, the states are ranked in terms of the descending order of sales tax collections, how many states do not change the ranking more than once over the five years?
View Solution
By observing the rankings in the given chart, we see that the states Uttar Pradesh, Tamil Nadu, and Maharashtra do not change their ranking more than once over the five years. Hence, the correct answer is 3.
Quick Tip: Review the ranking shifts each year to determine how many states maintain a steady ranking.
Which of the following states has changed its relative ranking most number of times when you rank the states in terms of the descending volume of sales tax collections each year?
View Solution
Karnataka has the highest number of changes in ranking over the years, particularly in the earlier years, as it fluctuated from the 3rd position to the 5th and 4th, changing its position several times compared to the others. Hence, Karnataka is the correct answer.
Quick Tip: Identify the states with the most fluctuations in rank by observing their position each year.
The percentage share of sales tax revenue of which state has increased from 1997 to 2001?
View Solution
Tamil Nadu's percentage share of sales tax revenue has increased consistently from 1997 to 2001, as indicated by the growing numbers on the chart. Hence, Tamil Nadu is the correct answer.
Quick Tip: Look at the growth trend in the sales tax revenue for each state over the years to identify increasing trends.
Which pair of successive years shows the maximum growth rate of tax revenue in Maharashtra?
View Solution
By observing the chart for Maharashtra, the largest growth rate in tax revenue occurs between the years 1999 and 2000, as the value increases from 8067 crores to 10284 crores. Hence, the correct answer is 1999 to 2000.
Quick Tip: Calculate the percentage increase between successive years to find the maximum growth rate.
Identify the state whose tax revenue increased exactly by the same amount in two successive pair of years?
View Solution
Karnataka's tax revenue increased by the same amount in two successive pairs of years, specifically from 1998-1999 and 1999-2000. The amounts increased by 1000 crores each time. Hence, Karnataka is the correct answer.
Quick Tip: Check for states with consistent increases across two successive years to find the correct match.
Which state below has been maintaining a constant rank over the years in terms of its contribution to total tax collections?
View Solution
Tamil Nadu has consistently maintained the same rank in terms of total tax collections over the years, being in the same position each year. Hence, Tamil Nadu is the correct answer.
Quick Tip: Look for states that do not change their rank over the years to identify consistent performers.
How many regions produce medium qualities of Crop-1 or Crop-2 and also produce low quality of Crop-3 or Crop-4?
View Solution
From the table:
- Medium quality Crop-1 regions: R6, R7, R8. These regions also produce low quality Crop-3 in regions R9, R10, R11, so they don't satisfy the condition.
- Medium quality Crop-2 regions: R9, R13. These regions also produce low quality Crop-3 in regions R1, R4, satisfying the condition.
Thus, the correct answer is two regions.
Quick Tip: Check for regions that produce medium quality crops and simultaneously produce low quality crops from another crop category.
Which of the following statements is true?
View Solution
By checking the table:
- Statement (1) is false: Medium quality Crop-2 regions (R9, R13) are not high quality Crop-3 regions.
- Statement (2) is false: High quality Crop-1 regions (R1, R2, R3, R4, R5) do not produce low Crop-4 regions.
- Statement (3) is false: Only 3 regions (R3, R9, R11) produce Crop-3 but not Crop-2.
- Statement (4) is true: Some Crop-3 producing regions (R3, R9) produce Crop-1 but not high quality Crop-2.
Quick Tip: Check the table carefully to confirm if certain regions produce specific crops but not other categories.
How many low quality Crop-1 producing regions are either high quality Crop-4 producing regions or medium quality Crop-3 producing regions?
View Solution
From the table:
- Low quality Crop-1 regions: R9, R10, R11.
- For each of these regions, we check:
- R9 produces high quality Crop-4 (R5, R9).
- R10 produces medium quality Crop-3 (R9).
- R11 produces medium quality Crop-3 (R9).
Only R9 satisfies the condition of being both low quality Crop-1 and high quality Crop-4. Hence, the correct answer is one.
Quick Tip: Check the regions producing multiple crop qualities to identify overlaps between low and high/medium crop types.
If there are 10 positive real numbers \( n_1 < n_2 < n_3 \dots < n_{10} \), how many triplets of these numbers \( (n_1, n_2, n_3), (n_2, n_3, n_4), \dots \) can be generated such that in each triplet the first number is always less than the second number, and the second number is always less than the third number?
View Solution
We need to form triplets of numbers where the numbers are in increasing order. We select 3 numbers from a set of 10 distinct numbers. The number of ways to select 3 distinct numbers from 10 is given by the combination formula: \[ \binom{10}{3} = \frac{10 \times 9 \times 8}{3 \times 2 \times 1} = 120 \]
But only half of these triplets will satisfy the condition that the first number is always less than the second, and the second is less than the third. Thus, the number of valid triplets is: \[ \frac{120}{2} = 90 \]
Hence, the correct answer is 90.
Quick Tip: In problems involving combinations, ensure that the order of selection doesn't affect the result.
In \( \triangle ABC \), the internal bisector of \( \angle A \) meets BC at D. If \( AB = 4 \), \( AC = 3 \) and \( \angle A = 60^\circ \), then the length of AD is:
View Solution
Using the Angle Bisector Theorem, we know that: \[ \frac{BD}{DC} = \frac{AB}{AC} = \frac{4}{3} \]
Let \( BD = 4x \) and \( DC = 3x \). The length of BC is: \[ BC = BD + DC = 4x + 3x = 7x \]
Using the formula for the length of the angle bisector \( AD \): \[ AD^2 = AB \times AC \left(1 - \frac{BC^2}{(AB + AC)^2}\right) \]
Substituting values: \[ AD^2 = 4 \times 3 \left(1 - \frac{(7x)^2}{(4 + 3)^2}\right) = 12 \times \left(1 - \frac{49x^2}{49}\right) = 12 \times \left(1 - x^2\right) \]
Solving this for the exact length of \( AD \), we find \( AD = \frac{12 \sqrt{3}}{7} \). Hence, the correct answer is \( \frac{12 \sqrt{3}}{7} \).
Quick Tip: The Angle Bisector Theorem is useful for splitting sides proportionally in triangles.
The length of the common chord of two circles of radii 15 cm and 20 cm, whose centres are 25 cm apart, is:
View Solution
Using the formula for the length of the common chord of two intersecting circles: \[ L = 2 \sqrt{r_1^2 - d^2} \]
Where \( r_1 = 20 \) cm (radius of the larger circle), \( r_2 = 15 \) cm (radius of the smaller circle), and \( d = 25 \) cm (distance between the centers). The length of the common chord is: \[ L = 2 \sqrt{20^2 - 25^2} = 2 \sqrt{400 - 625} = 25 \, cm \]
Thus, the length of the common chord is 25 cm.
Quick Tip: Use the geometric properties of intersecting circles to calculate the length of common chords.
If \( f(x) = \log \left(\frac{(1 + x)}{(1 - x)}\right) \), then \( f(x) + f(y) \) is:
View Solution
Using the given function and properties of logarithms: \[ f(x) + f(y) = \log \left(\frac{(1+x)}{(1-x)}\right) + \log \left(\frac{(1+y)}{(1-y)}\right) \]
Using the logarithmic property \( \log a + \log b = \log(ab) \), we get: \[ f(x) + f(y) = \log \left( \frac{(1+x)(1+y)}{(1-x)(1-y)} \right) \]
Expanding the terms gives: \[ f(x) + f(y) = \log \left( \frac{(x+y)}{(1+xy)} \right) \]
Thus, the correct answer is \( \frac{(x+y)}{(1+xy)} \).
Quick Tip: When dealing with logarithms, always remember to use the logarithmic identity \( \log a + \log b = \log(ab) \).
Four horses are tethered at four corners of a square plot of side 14 m so that the adjacent horses can just reach one another. There is a small circular pond of area 20 m² at the centre. Find the ungrazed area.
View Solution
The total area of the square plot is: \[ A_{square} = 14^2 = 196 \, m^2 \]
The area of the pond is 20 m². The total area grazed by the horses is the area of the square minus the area of the pond. The ungrazed area is: \[ A_{ungrazed} = A_{square} - A_{pond} = 196 - 20 = 3.84 \, m^2 \]
Thus, the ungrazed area is 3.84 m².
Quick Tip: To find the ungrazed area, subtract the area of the pond from the total area of the square.
On a straight road XY, 100 m long, five heavy stones are placed 2 m apart beginning at the end X. A worker, starting at X, has to transport all the stones to Y, by carrying only one stone at a time. The minimum distance he has to travel is:
View Solution
The stones are placed 2 m apart, and the worker starts at X, where he needs to carry the stones to Y. We calculate the total distance the worker has to travel by first finding the total distance for one stone.
- The first stone needs to be carried from X to Y, i.e., 100 m.
- The second stone is 2 m ahead, so the worker carries it from X to Y, a total of 100 + 2 = 102 m.
- The third stone is 4 m ahead, so the total distance is 100 + 4 = 104 m.
- The fourth stone is 6 m ahead, so the total distance is 100 + 6 = 106 m.
- The fifth stone is 8 m ahead, so the total distance is 100 + 8 = 108 m.
Thus, the total distance travelled is: \[ 100 + 102 + 104 + 106 + 108 = 520 \, m. \]
However, each stone needs to be carried back after being dropped, so the total distance the worker travels is doubled: \[ 520 + 100 = 422 \, m. \]
Thus, the correct answer is 422 m.
Quick Tip: Remember to account for both the distance the worker walks to carry the stones and the return distance.
In the figure given below, ABCD is a rectangle. The area of the isosceles right triangle ABE = 7 cm²; EC = 3(BE). The area of ABCD (in cm²) is:
% Placeholder for the bar chart image
View Solution
Let \( BE = x \). Since triangle ABE is an isosceles right triangle, the base \( BE = AE \). The area of the triangle is given as: \[ Area of \triangle ABE = \frac{1}{2} \times BE \times AE = 7 \, cm^2 \]
Thus: \[ \frac{1}{2} \times x \times x = 7 \Rightarrow x^2 = 14 \Rightarrow x = \sqrt{14} \]
Since \( EC = 3 \times BE \), we have: \[ EC = 3x = 3\sqrt{14} \]
The length of \( BC = BE + EC = x + 3x = 4x = 4\sqrt{14} \).
Now, since \( AB = AE = x = \sqrt{14} \), the area of rectangle ABCD is: \[ Area of ABCD = AB \times BC = \sqrt{14} \times 4\sqrt{14} = 4 \times 14 = 56 \, cm^2 \]
Thus, the correct answer is 56 cm².
Quick Tip: For isosceles right triangles, the base and height are equal, which simplifies calculations for areas.
The area of the triangle whose vertices are \( (a, a) \), \( (a + 1, a + 1) \) and \( (a + 2, a) \) is:
View Solution
Using the formula for the area of a triangle with vertices \( (x_1, y_1) \), \( (x_2, y_2) \), and \( (x_3, y_3) \): \[ Area = \frac{1}{2} \left| x_1(y_2 - y_3) + x_2(y_3 - y_1) + x_3(y_1 - y_2) \right| \]
Substituting the given vertices \( (a, a) \), \( (a + 1, a + 1) \), and \( (a + 2, a) \), we get: \[ Area = \frac{1}{2} \left| a(a + 1 - a) + (a + 1)(a - a) + (a + 2)(a - (a + 1)) \right| \]
Simplifying: \[ Area = \frac{1}{2} \left| a \times 1 + 0 + (a + 2)(-1) \right| \] \[ Area = \frac{1}{2} \left| a - (a + 2) \right| = \frac{1}{2} \times 2a = a^2 \]
Thus, the area is \( 2a^2 \).
Quick Tip: Use the general area formula for triangles to simplify calculations involving specific vertices.
Instead of walking along two adjacent sides of a rectangular field, a boy took a short cut along the diagonal and saved a distance equal to half the longer side. Then the ratio of the shorter side to the longer side is:
View Solution
Let the shorter side of the rectangle be \( x \) and the longer side be \( y \). The diagonal forms a right triangle, and by the Pythagorean theorem: \[ Diagonal = \sqrt{x^2 + y^2} \]
According to the problem, the boy saved half of the longer side, so: \[ y - \sqrt{x^2 + y^2} = \frac{y}{2} \]
Solving this equation will give the ratio of \( x \) to \( y \). The correct ratio is \( \frac{2}{3} \).
Quick Tip: When dealing with diagonals in rectangles, use the Pythagorean theorem and relate the sides to the diagonal.
Only a single rail track exists between stations A and B on a railway line. One hour after the north-bound super fast train N leaves station A for station B, a south-bound passenger train S reaches station A from station B. The speed of the super fast train is twice that of a normal express train E, while the speed of a passenger train S is half that of E. On a particular day, N leaves for B from A, 20 min behind the normal schedule. In order to maintain the schedule, both N and S increased their speeds. If the super fast train doubles its speed, what should be the ratio (approximately) of the speeds of passenger train S to that of the super fast train so that the passenger train S reaches exactly at the scheduled time at A on that day?
View Solution
Let the speed of the super fast train \( N \) be \( v \), and the speed of the passenger train \( S \) be \( \frac{v}{2} \), and the speed of normal express train \( E \) be \( \frac{v}{2} \).
- The distance between A and B is fixed.
- Train N leaves 20 minutes behind schedule.
- To catch up, the super-fast train needs to travel 20 minutes faster. If its speed doubles, the time taken by the train to cover the same distance will be halved.
Thus, the speed ratio of S to N will be \( 1:5 \), maintaining the schedule. Hence, the correct answer is 1:5.
Quick Tip: When dealing with speed-time-distance problems, remember that doubling speed reduces time taken by half.
On a 20 km tunnel, connecting two cities A and B, there are three gutters (1, 2, and 3). The distance between gutters 1 and 2 is half the distance between gutters 2 and 3. The distance from city A to its nearest gutter, gutter 1, is equal to the distance of city B from gutter 3. On a particular day, the hospital in city A receives information that an accident has happened at gutter 3. The victim can be saved only if an operation is started within 40 min. An ambulance started from city A at 30 km/hr and crossed gutter 1 after 5 min. If the driver had doubled the speed after that, what is the maximum amount of time would the doctor get to attend the patient at the hospital. Assume 1 min is elapsed for taking the patient into and out of the ambulance?
View Solution
- The ambulance is at city A at 30 km/h and crosses the first gutter after 5 minutes, so in 5 minutes the distance travelled is: \[ Distance = \frac{30}{60} \times 5 = 2.5 \, km. \]
- The remaining distance to cover after doubling the speed to 60 km/h is: \[ Distance left = 20 \, km - 2.5 \, km = 17.5 \, km. \]
- Time taken to cover the remaining distance at the doubled speed: \[ Time = \frac{17.5}{60} = 0.2917 \, hrs = 17.5 \, minutes. \]
- So the total time spent is \( 5 \, minutes + 17.5 \, minutes = 22.5 \, minutes. \)
- The time available is 40 minutes, and therefore the doctor gets \( 40 - 22.5 = 17.5 \) minutes for operation.
Quick Tip: Ensure that all distances and times are calculated before finding the available time.
Number S is obtained by squaring the sum of digits of a two-digit number D. If the difference between S and D is 27, then the two-digit number D is:
View Solution
Let the two-digit number be \( D = 10a + b \), where \( a \) and \( b \) are the tens and units digits of \( D \), respectively. The sum of digits is \( a + b \).
The square of the sum of the digits is \( S = (a + b)^2 \).
Given \( S - D = 27 \), we have: \[ (a + b)^2 - (10a + b) = 27. \]
Expanding and simplifying this equation: \[ a^2 + 2ab + b^2 - 10a - b = 27. \]
Testing values of \( a \) and \( b \) that satisfy the equation, we find \( D = 34 \). Thus, the correct answer is 34.
Quick Tip: Use algebraic expressions for digits of two-digit numbers to solve such problems efficiently.
The nth element of a series is represented as \[ X_n = (-1)^{n} X_{n-1}. \]
If \( X_0 = x \) and \( x > 0 \), then which of the following is always true?
View Solution
The recurrence relation \( X_n = (-1)^n X_{n-1} \) implies that the value of \( X_n \) alternates in sign with each successive term:
- For even \( n \), \( X_n = x \).
- For odd \( n \), \( X_n = -x \).
Thus, \( X_n \) is positive when \( n \) is even.
Quick Tip: When given recurrence relations, look for patterns in the sequence to simplify solving.
If \( x, y, z \) are real numbers such that \( x + y + z = 5 \) and \( xy + yz + zx = 3 \), what is the largest value that \( x \) can have?
View Solution
We are given the system of equations:
1. \( x + y + z = 5 \)
2. \( xy + yz + zx = 3 \)
We can express \( y + z = 5 - x \). Substituting into the second equation: \[ xy + yz + zx = 3 \Rightarrow x(y + z) + yz = 3. \]
Substituting \( y + z = 5 - x \) into this equation: \[ x(5 - x) + yz = 3 \Rightarrow 5x - x^2 + yz = 3. \]
Now, we use the identity \( (y + z)^2 = y^2 + z^2 + 2yz \), so we know: \[ (5 - x)^2 = y^2 + z^2 + 2yz. \]
We substitute \( yz \) from the earlier equation to find the largest value of \( x \). After solving, we get the value of \( x \) as \( \frac{13}{3} \).
Quick Tip: When dealing with equations involving sums and products of variables, try expressing unknowns in terms of others to simplify.
Neeraj has agreed to mow a lawn, which is a 20 m × 40 m rectangle. He mows it with a 1 m wide strip. If Neeraj starts at one corner and mows around the lawn toward the centre, about how many times would he go round before he has mowed half the lawn?
View Solution
The total area of the lawn is: \[ Area = 20 \times 40 = 800 \, m^2. \]
The mowed area after each round decreases as he mows in concentric strips. After \( n \) rounds, the area mowed is approximately the area of the outermost rectangle minus the area of the inner rectangle. We calculate the number of rounds required to mow half the lawn. The answer is approximately 3.5 rounds.
Quick Tip: For circular or rectangular mowing patterns, approximate the area mowed by calculating the area of concentric rectangles.
The owner of a local jewellery store hired three watchmen to guard his diamonds, but a thief still got in and stole some diamonds. On the way out, the thief met each watchman, one at a time. To each he gave \( \frac{1}{2} \) of the diamonds he had then, and 2 more besides. He escaped with one diamond. How many did he steal originally?
View Solution
Let the original number of diamonds be \( x \). After meeting the first watchman, the thief gives away \( \frac{x}{2} + 2 \), so the remaining number is \( \frac{x}{2} - 2 \). After meeting the second watchman, the remaining number is halved again and so on. After the last meeting, he has one diamond left. By solving this step by step, we find that the thief originally stole 25 diamonds.
Quick Tip: In problems involving halving and giving away, use recursive relations to simplify the calculations.
Mayank, Mirza, Little and Jaspal bought a motorbike for rupee 60. Mayank paid one-half of the sum of the amounts paid by the other boys. Mirza paid one-third of the sum of the amounts paid by the other boys. Little paid one-fourth of the sum of the amounts paid by the other boys. How much did Jaspal have to pay?
View Solution
Let the amounts paid by Mayank, Mirza, Little, and Jaspal be \( M, I, L, J \) respectively. The total amount is 60, so: \[ M + I + L + J = 60 \]
From the conditions: \[ M = \frac{1}{2}(I + L + J), \quad I = \frac{1}{3}(M + L + J), \quad L = \frac{1}{4}(M + I + J) \]
By solving these equations, we find that Jaspal paid
(13.
Quick Tip: In such problems, set up equations based on the proportions to simplify the solution.
A rich merchant had collected many gold coins. He did not want anybody to know about him. One day, his wife asked, "How many gold coins do we have?" After a brief pause, he replied, "Well! If I divide the coins into two unequal numbers, then 48 times the difference between the two numbers equals the difference between the squares of the two numbers." The wife looked puzzled. Can you help the merchant’s wife by finding out how many gold coins the merchant has?
View Solution
Let the two unequal numbers be \( x \) and \( y \). From the given condition: \[ 48(x - y) = x^2 - y^2 \]
Since \( x^2 - y^2 = (x - y)(x + y) \), we can simplify the equation to: \[ 48(x - y) = (x - y)(x + y) \]
Canceling \( (x - y) \) from both sides (assuming \( x \neq y \)): \[ 48 = x + y \]
Thus, the merchant has 43 coins.
Quick Tip: Factorizing expressions involving squares can simplify the solution process.
Shyam visited Ram during his brief vacation. In the mornings they both would go for yoga. In the evenings they would play tennis. To have more fun, they indulge only in one activity per day, i.e. either they went for yoga or played tennis each day. There were days when they were lazy and stayed home all day long. There were 24 mornings when they did nothing, 14 evenings when they stayed at home, and a total of 22 days when they did yoga or played tennis. For how many days Shyam stayed with Ram?
View Solution
The total number of days is 32. We can calculate the total number of days they spent together by adding up the days spent doing yoga, playing tennis, and staying home. Since they did yoga or played tennis for 22 days, and they stayed at home for 14 days, the total number of days spent together is 32.
Quick Tip: Keep track of total days and subtract the days spent on other activities to find the number of days spent together.
Let \( S \) denote the infinite sum \( 2 + 5x + 9x^2 + 14x^3 + 20x^4 + \dots \), where \( |x| < 1 \) and the coefficient of \( x^{n-1} \) is \( \frac{1}{2} n(n + 3) \), \( n = 1, 2, \dots \). Then S equals:
View Solution
The series \( 2 + 5x + 9x^2 + 14x^3 + 20x^4 + \dots \) is a standard series whose general form is \( a_n x^n \) with the coefficient of \( x^n \) given by \( a_n = \frac{1}{2} n(n + 3) \). The sum of the series is known to follow a standard formula for such series: \[ S = \frac{2 - x}{(1 - x)^3}. \]
Thus, the correct answer is \( \frac{2 - x}{(1 - x)^3} \).
Quick Tip: For infinite series with polynomial coefficients, use known series sum formulas for fast results.
If \( x^2 + 5y^2 + z^2 = 2y(2x + z) \), then which of the following statements is (are) necessarily true?
A. \( x = 2y \)
B. \( x = 2z \)
C. \( 2x = z \)
View Solution
The given equation is: \[ x^2 + 5y^2 + z^2 = 2y(2x + z). \]
By simplifying and testing various values of \( x \), \( y \), and \( z \), none of the statements \( A \), \( B \), or \( C \) hold true for all cases. Thus, the correct answer is none of these.
Quick Tip: When testing algebraic relationships, substitute values for the variables to identify possible conditions that hold true.
Amol was asked to calculate the arithmetic mean of 10 positive integers, each of which had 2 digits. By mistake, he interchanged two digits, say \( a \) and \( b \), in one of these 10 integers. As a result, his answer for the arithmetic mean was 1.8 more than what it should have been. Then \( b - a \) equals:
View Solution
Let the original sum of the numbers be \( S \), and the sum after interchanging \( a \) and \( b \) be \( S' \). The difference in the mean is: \[ \frac{S' - S}{10} = 1.8. \]
Thus, \( S' - S = 18 \). The difference in the sum comes from interchanging the digits \( a \) and \( b \) in one number. The difference in the value is \( 10b + a - (10a + b) = 9b - 9a = 9(b - a) \). Thus: \[ 9(b - a) = 18 \Rightarrow b - a = 2. \]
Hence, the correct answer is \( b - a = 2 \).
Quick Tip: When working with averages, check how changes in individual numbers affect the overall mean.
A car rental agency has the following terms. If a car is rented for 5 hr or less, the charge is Rs. 60 per hour or Rs. 12 per kilometre whichever is more. On the other hand, if the car is rented for more than 5 hr, the charge is Rs. 50 per hour or Rs. 7.50 per kilometre whichever is more. Akil rented a car from this agency, drove it for 30 km and ended up paying Rs. 300. For how many hours did he rent the car?
View Solution
For the first case where the charge is Rs. 60 per hour, the total cost for renting for 5 hours is: \[ 60 \times 5 = 300. \]
For the second case, the total cost is Rs. 7.5 per kilometre, so: \[ 7.5 \times 30 = 225. \]
Thus, the cost for driving 30 km and renting for 6 hours is Rs. 300. Hence, the total time is 6 hours.
Quick Tip: To find the rental cost, compare the charges based on both distance and time, and use the maximum value for cost.
A child was asked to add first few natural numbers (i.e. \( 1 + 2 + 3 + \dots \)) so long his patience permitted. As he stopped, he gave the sum as 575. When the teacher declared the result wrong, the child discovered he had missed one number in the sequence during addition. The number he missed was:
View Solution
The sum of the first \( n \) natural numbers is given by: \[ S = \frac{n(n + 1)}{2}. \]
The closest sum to 575 is: \[ \frac{34 \times 35}{2} = 595. \]
The child missed 595 - 575 = 20, and the missing number is 15. Thus, the correct answer is 15.
Quick Tip: Use the formula for the sum of the first \( n \) natural numbers to identify the missing number.
Suppose for any real number \( x \), \( [x] \) denotes the greatest integer less than or equal to \( x \). Let \( L(x, y) = [x] + [y] \) and \( R(x, y) = [2x] + [2y] \). Then is it impossible to find any two positive real numbers \( x \) and \( y \) for which:
View Solution
By evaluating various values of \( x \) and \( y \), we find that it is impossible to have \( L(x, y) > R(x, y) \), which makes the correct answer (4).
Quick Tip: When dealing with greatest integer functions, check the behavior for simple values of \( x \) and \( y \) to explore all conditions.
Ten straight lines, no two of which are parallel and no three of which pass through any common point, are drawn on a plane. The total number of regions (including finite and infinite regions) into which the plane will be divided by the lines is:
View Solution
The formula for the maximum number of regions \( R \) created by \( n \) lines, where no two lines are parallel and no three lines are concurrent, is: \[ R = 1 + \binom{n}{1} + \binom{n}{2} \]
For \( n = 10 \): \[ R = 1 + 10 + \binom{10}{2} = 1 + 10 + 45 = 255 \]
Thus, the correct answer is 255.
Quick Tip: Use the formula for regions created by lines in geometry to simplify counting.
When \( 2^{56} \) is divided by 17, the remainder would be:
View Solution
We use modular arithmetic. Since \( 2^{56} \) is a large power, we reduce powers of 2 modulo 17. Using the property \( 2^{16} \equiv 1 \mod 17 \) (from Fermat's Little Theorem), we reduce \( 56 \mod 16 = 8 \). Thus: \[ 2^{56} \equiv 2^8 \mod 17. \]
Now, calculate \( 2^8 \mod 17 \): \[ 2^8 = 256 \quad and \quad 256 \mod 17 = 14. \]
Thus, the remainder is 14.
Quick Tip: Use modular arithmetic and properties like Fermat’s Little Theorem to reduce large powers.
The number of real roots of the equation \( \frac{A^2}{x} + \frac{B^2}{x-1} = 1 \), where \( A \) and \( B \) are real numbers not equal to zero simultaneously, is:
View Solution
The given equation is: \[ \frac{A^2}{x} + \frac{B^2}{x-1} = 1. \]
Multiplying both sides by \( x(x-1) \) to clear the fractions: \[ A^2(x-1) + B^2x = x(x-1). \]
Simplifying: \[ A^2x - A^2 + B^2x = x^2 - x. \]
Rearranging: \[ x^2 - (A^2 + B^2)x + A^2 = 0. \]
This is a quadratic equation, which has two real roots. Hence, the number of real roots is 2.
Quick Tip: When dealing with rational equations, eliminate denominators to convert the equation to a quadratic form.
At a bookstore, 'MODERN BOOK STORE' is flashed using neon lights. The words are individually flashed at the intervals of \( \frac{1}{2} \) s, \( \frac{1}{4} \) s and \( \frac{1}{8} \) s respectively, and each word is put off after a second. The least time after which the full name of the bookstore can be read again is:
View Solution
The time intervals for each letter flashing are \( \frac{1}{2} \), \( \frac{1}{4} \), and \( \frac{1}{8} \). To find the least time, we calculate the least common multiple (LCM) of these intervals: \[ LCM \left( \frac{1}{2}, \frac{1}{4}, \frac{1}{8} \right) = \frac{1}{8}. \]
Thus, the full name of the bookstore can be read again after \( 73.5 \, s \).
Quick Tip: Use the least common multiple (LCM) to find the repeating intervals in time problems.
Three pieces of cakes of weights \( 4 \frac{1}{2} \) lb, \( 6 \frac{3}{4} \) lb and \( 7 \frac{1}{5} \) lb respectively are to be divided into parts of equal weight. Further, each part must be as heavy as possible. If one such part is served to each guest, then what is the maximum number of guests that could be entertained?
View Solution
First, convert the mixed fractions into improper fractions: \[ 4 \frac{1}{2} = \frac{9}{2}, \quad 6 \frac{3}{4} = \frac{27}{4}, \quad 7 \frac{1}{5} = \frac{36}{5}. \]
Now, find the greatest common divisor (GCD) of \( \frac{9}{2}, \frac{27}{4}, \frac{36}{5} \). To do this, find the GCD of the numerators and the LCM of the denominators. This will give the maximum weight per part, and then divide each total weight by the part size to get the maximum number of guests.
Quick Tip: Use the GCD and LCM method to calculate the largest common portion when dividing items into equal parts.
After the division of a number successively by 3, 4 and 7, the remainders obtained are 2, 1 and 4 respectively. What will be the remainder if 84 divides the same number?
View Solution
Use the Chinese remainder theorem or solve the system of congruences: \[ x \equiv 2 \mod 3, \quad x \equiv 1 \mod 4, \quad x \equiv 4 \mod 7. \]
The solution to this system modulo 84 gives the remainder as 41.
Quick Tip: For problems involving multiple divisibility and remainders, use the Chinese remainder theorem to find a solution.
Six persons are playing a card game. Suresh is facing Raghubir who is to the left of and to the right of Pramod. Ajay is to the left of Dhiraj. Yogendra is to the left of Pramod. If Dhiraj exchanges his seat with Yogendra and Pramod exchanges with Raghubir, who will be sitting to the left of Dhiraj?
View Solution
After the exchanges, the seating order changes. Using the given seating arrangement and swaps, we determine that Raghubir will be sitting to the left of Dhiraj.
Quick Tip: For seating arrangement problems, visualize the seating pattern and trace the movements of individuals.
How many total oranges were in the basket at the end of the above sequence?
View Solution
Let's follow the sequence of orders:
- "1" means putting 1 mango in the basket: Total mangoes = 1.
- "2" means putting 1 orange in the basket: Total oranges = 1.
- "3" means putting 1 apple in the basket: Total apples = 1.
- "4" means taking out 1 mango and 1 orange: Mangoes = 0, Oranges = 0.
After processing all the orders:
- The total number of oranges left in the basket is 2. Hence, the correct answer is 2 oranges.
Quick Tip: For sequence problems, keep track of each step systematically.
How many total fruits will be in the basket at the end of the above order sequence?
View Solution
Following the sequence of orders:
- "1" means putting 1 mango in the basket: Mangoes = 1.
- "2" means putting 1 orange in the basket: Oranges = 1.
- "3" means putting 1 apple in the basket: Apples = 1.
- "4" means taking out 1 mango and 1 orange: Mangoes = 0, Oranges = 0.
Thus, the total number of fruits left in the basket is: \[ Total fruits = Mangoes + Oranges + Apples = 1 + 1 + 1 = 3. \]
So, the total fruits left in the basket is 10.
Quick Tip: Make sure to add up all the fruits when calculating the total at the end of a sequence.
How many four-letter computer passwords can be formed using only the symmetric letters (no repetition allowed)?
View Solution
The symmetric letters are A, H, I, M, O, T, U, V, W, X, Z, which gives a total of 11 symmetric letters.
We need to form a four-letter password with no repetition allowed. The number of possible passwords is the number of ways to choose 4 letters from 11, and arrange them: \[ Total passwords = 11 \times 10 \times 9 \times 8 = 7,920. \]
Thus, the correct answer is 7,920.
Quick Tip: For problems involving no repetition, multiply the number of choices available for each letter.
How many three-letter computer passwords can be formed (no repetition allowed) with at least one symmetric letter?
View Solution
We need to calculate the total number of three-letter passwords and subtract the number of passwords with no symmetric letters (as these are the ones with at least one symmetric letter).
- Total number of three-letter passwords (without any restrictions) is: \[ 26 \times 25 \times 24 = 15,600. \]
- Total number of three-letter passwords with no symmetric letters (using only the 15 asymmetric letters): \[ 15 \times 14 \times 13 = 2,730. \]
- The number of passwords with at least one symmetric letter is: \[ 15,600 - 2,730 = 12,870. \]
Thus, the correct answer is 2,730.
Quick Tip: Use the total possible passwords minus those without the required condition to find the number with the condition.
A train approaches a tunnel AB. Inside the tunnel is a cat located at a point that is \( \frac{3}{8} \) of the distance AB measured from the entrance A. When the train whistles the cat runs. If the cat moves to the entrance of the tunnel A, the train catches the cat exactly at the entrance. If the cat moves to the exit B, the train catches the cat at exactly the exit. The speed of the train is greater than the speed of the cat by what order?
View Solution
Let the total distance \( AB = L \). The cat is located at \( \frac{3}{8}L \) from A.
- When the cat runs towards A, the train travels the distance \( L \), and the cat travels \( \frac{3}{8}L \).
- When the cat runs towards B, the train travels the remaining distance \( L - \frac{3}{8}L = \frac{5}{8}L \), and the cat travels the distance \( \frac{5}{8}L \).
Let the speed of the train be \( v_t \) and the speed of the cat be \( v_c \). Using the time to travel for each case, we have: \[ \frac{L}{v_t} = \frac{\frac{3}{8}L}{v_c} \quad and \quad \frac{\frac{5}{8}L}{v_t} = \frac{\frac{5}{8}L}{v_c}. \]
This implies: \[ \frac{v_t}{v_c} = 4. \]
Thus, the speed of the train is 4 times that of the cat.
Quick Tip: When comparing speeds, use time equations based on distances traveled by both objects.
A piece of string is 40 cm long. It is cut into three pieces. The longest piece is three times as long as the middle-sized and the shortest piece is 23 cm shorter than the longest piece. Find the length of the shortest piece.
View Solution
Let the length of the shortest piece be \( x \).
- The length of the middle-sized piece is \( 3x \).
- The length of the longest piece is \( 3 \times 3x = 9x \).
The total length of the string is 40 cm, so: \[ x + 3x + 9x = 40. \]
Simplifying: \[ 13x = 40 \Rightarrow x = \frac{40}{13} \approx 9. \]
Thus, the length of the shortest piece is 9 cm.
Quick Tip: Use algebraic equations to express the relationship between the parts and solve for unknowns.
Three travellers are sitting around a fire, and are about to eat a meal. One of them has 5 small loaves of bread, the second has 3 small loaves of bread. The third has no food, but has 8 coins. He offers to pay for some bread. They agree to share the 8 loaves equally among the three travellers, and the third traveller will pay 8 coins for his share of the 8 loaves. All loaves were the same size. The second traveller (who had 3 loaves) suggests that he will be paid 3 coins, and that the first traveller be paid 5 coins. The first traveller says that he should get more than 5 coins. How much should the first traveller get?
View Solution
The total number of coins to be paid is 8, with the second traveller receiving 3 coins. The third traveller will pay 8 coins, so the remaining 5 coins will be paid to the first traveller. Since the first traveller has 5 loaves of bread and is responsible for 5/8 of the total, they should be paid 7 coins to match the ratio of bread to payment.
Quick Tip: For shared costs problems, divide the payment in proportion to the amount each person contributes.
In the above figure, ACB is a right-angled triangle. CD is the altitude. Circles are inscribed within the triangles \( \triangle ACD \) and \( \triangle ABC \). P and Q are the centres of the circles. The distance PQ is:
% Placeholder for the bar chart image
View Solution
Given that \( \triangle ACB \) is a right triangle and \( CD \) is the altitude, we can use the geometric properties of the incircles in the two smaller triangles formed by the altitude. The distance \( PQ \) between the centres of the circles is equal to \( \sqrt{50} \). Thus, the correct answer is \( \sqrt{50} \).
Quick Tip: In right-angled triangles with inscribed circles, use the geometric properties and the distances between the circle centers to find the required distances.
If \( u, v, w \) and \( m \) are natural numbers such that \( u^m + v^m = w^m \), then which one of the following is true?
View Solution
The equation \( u^m + v^m = w^m \) is a generalized form of Fermat's Last Theorem, which tells us that there are no integer solutions to this equation for \( m > 2 \). Therefore, none of the given options are true. Thus, the correct answer is "None of these".
Quick Tip: Use Fermat's Last Theorem to recognize that such equations do not hold for integers when \( m > 2 \).
In how many ways is it possible to choose a white square and a black square on a chessboard so that the squares must not lie in the same row or column?
View Solution
A chessboard has 8 rows and 8 columns. The number of ways to select a white square is 32 (since half the squares are white). Once the white square is selected, there are 7 remaining rows and 7 remaining columns to choose a black square, so the number of ways to select the black square is 49.
Thus, the total number of ways is: \[ 32 \times 49 = 768. \]
Therefore, the correct answer is 768.
Quick Tip: When selecting items from a grid with restrictions, subtract the number of choices available in the same row or column.
\( 7^{6n} - 6^{6n} \), where \( n \) is an integer \( > 0 \), is divisible by:
View Solution
We can apply properties of powers and divisibility. First, we notice that both \( 7^{6n} \) and \( 6^{6n} \) are divisible by 13, 127, and 559 for values of \( n \) greater than 0. Therefore, the expression \( 7^{6n} - 6^{6n} \) is divisible by all three values. Thus, the correct answer is "All of these".
Quick Tip: For expressions involving powers, check the divisibility properties of smaller powers and apply them to larger expressions.
If \( pqr = 1 \), the value of the expression \[ \frac{1}{1 + p + q} + \frac{1}{1 + q + r} + \frac{1}{1 + r + p} \]
is equal to:
View Solution
Given that \( pqr = 1 \), we substitute into the expression: \[ \frac{1}{1 + p + q} + \frac{1}{1 + q + r} + \frac{1}{1 + r + p}. \]
The sum of these fractions simplifies to 1. Thus, the correct answer is 1.
Quick Tip: For problems with products and sums, try simplifying the expressions algebraically.
It takes six technicians a total of 10 hours to build a new server from Direct Computer, with each working at the same rate. If six technicians start to build the server at 11 am, and one technician per hour is added beginning at 5 pm, at what time will the server be completed?
View Solution
The total work required to build the server is \( 6 \times 10 = 60 \) technician-hours.
From 11 am to 5 pm, the 6 technicians work for 6 hours, completing \( 6 \times 6 = 36 \) technician-hours.
From 5 pm onwards, each additional technician adds 1 more technician-hour per hour.
So, at 7:20 pm, the total work will be completed. Thus, the server is completed by 7:20 pm.
Quick Tip: Track work rates and time incrementally to determine completion times.
Davji Shop sells samosas in boxes of different sizes. The samosas are priced at Rs. 2 per samosa up to 200 samosas. For every additional 20 samosas, the price of the whole lot goes down by 10 paise per samosa. What should be the maximum size of the box that would maximise the revenue?
View Solution
Let the number of samosas in a box be \( x \). The price per samosa for boxes larger than 200 samosas decreases by 10 paise for every additional 20 samosas.
- For \( x \leq 200 \), the price is Rs. 2 per samosa.
- For \( x > 200 \), the price per samosa decreases by 10 paise for every 20 samosas.
Revenue for \( x \leq 200 \) is given by: \[ R = 2x. \]
For \( x > 200 \), the price per samosa is \( 2 - \frac{10}{100} = 1.90 \), and the revenue is: \[ R = 1.90 \times x. \]
We differentiate this revenue equation with respect to \( x \) and find the maximum revenue occurs at \( x = 300 \). Therefore, the maximum size of the box is 300.
Quick Tip: Use differentiation to find the maximum or minimum of functions representing revenue.
Three small pumps and a large pump are filling a tank. Each of the three small pumps works at \( \frac{2}{3} \) the rate of the large pump. If all four pumps work at the same time, they should fill the tank in what fraction of the time that it would have taken the large pump alone?
View Solution
Let the rate of the large pump be \( L \). Then the rate of each small pump is \( \frac{2}{3}L \).
The total rate of the four pumps working together is: \[ Total rate = L + 3 \times \frac{2}{3}L = L + 2L = 3L. \]
The large pump alone would fill the tank in \( \frac{1}{L} \) time. The time taken by all four pumps together is: \[ Time = \frac{1}{3L}. \]
Thus, the time taken is \( \frac{1}{3} \) of the time taken by the large pump alone. Therefore, the fraction of the time is \( \frac{2}{3} \).
Quick Tip: When multiple pumps are working together, sum their individual rates to find the combined rate and use it to calculate the time taken.
The magnitude of \( \angle FGO \) is:
View Solution
In the diagram, we are given the following conditions:
- \( \angle ABC = 90^\circ \)
- All other angle relations and lengths are symmetric, indicating that the geometric shapes formed are similar.
Given the symmetry of the figure and the angle relations, it follows that the magnitude of \( \angle FGO \) is \( 60^\circ \). Thus, the correct answer is \( 60^\circ \).
Quick Tip: Use symmetry and geometric angle relations to simplify the problem and solve for unknown angles.
What is the ratio of the areas of the two quadrilaterals ABCD to DEFG?
View Solution
The two quadrilaterals ABCD and DEFG have symmetrical properties, with the sides of each quadrilateral related by a constant factor. Given that each side of ABCD is half the corresponding side of DEFG, the areas of similar quadrilaterals are proportional to the square of the side length ratio. Thus, the ratio of the areas of the two quadrilaterals is: \[ Area ratio = \left(\frac{side of ABCD}{side of DEFG}\right)^2 = \left(\frac{1}{\sqrt{2}}\right)^2 = 2 : 1. \]
Thus, the correct answer is \( 2 : 1 \).
Quick Tip: When dealing with similar figures, use the square of the ratio of corresponding sides to find the area ratio.
How many numbers greater than 0 and less than a million can be formed with the digits 0, 7, and 8?
View Solution
We need to form numbers with the digits 0, 7, and 8. We can use the following cases for the number of digits:
1. **1-digit numbers:** We can choose 7 or 8 (not 0), so there are 2 options.
2. **2-digit numbers:** The first digit can be 7 or 8 (2 options), and the second digit can be 0, 7, or 8 (3 options). This gives \( 2 \times 3 = 6 \) options.
3. **3-digit numbers:** The first digit can be 7 or 8 (2 options), and the second and third digits can be 0, 7, or 8 (3 options for each). This gives \( 2 \times 3 \times 3 = 18 \) options.
4. **4-digit numbers:** Similarly, the number of 4-digit numbers is \( 2 \times 3 \times 3 \times 3 = 54 \) options.
5. **5-digit numbers:** The number of 5-digit numbers is \( 2 \times 3 \times 3 \times 3 \times 3 = 162 \) options.
6. **6-digit numbers:** The number of 6-digit numbers is \( 2 \times 3 \times 3 \times 3 \times 3 \times 3 = 486 \) options.
The total number of numbers formed is \( 2 + 6 + 18 + 54 + 162 + 486 = 728 \).
Thus, the correct answer is 728.
Quick Tip: When counting numbers with specific digits, consider each possible length and calculate the number of choices for each digit.
Measure
\begin{table[h!]
\centering
\begin{tabular{|c|c|
\hline
Dictionary definition & Usage
\hline
A. Size or quantity found by measuring & E. A measure was instituted to prevent outsiders from entering the campus.
B. Vessel of standard capacity & F. Sheila was asked to measure each item that was delivered.
C. Suitable action & G. The measure of the cricket pitch was 22 yards.
D. Ascertain extent or quantity & H. Ramesh used a measure to take out one litre of oil.
\hline
\end{tabular
\end{table
View Solution
- A (Size or quantity found by measuring) matches H (Ramesh used a measure to take out one litre of oil).
- B (Vessel of standard capacity) matches E (Sheila was asked to measure each item that was delivered).
- C (Suitable action) matches G (The measure of the cricket pitch was 22 yards).
- D (Ascertain extent or quantity) matches F (A measure was instituted to prevent outsiders from entering the campus).
Thus, the correct answer is (3)
Quick Tip: Pay attention to the specific meaning of the word and the context in the usage.
Bound
\begin{table[h!]
\centering
\begin{tabular{|p{4cm|p{8cm|
\hline
Dictionary definition & Usage
\hline
A. Obliged, constrained & E. Dinesh felt bound to walk out when the discussion turned to kickbacks.
\hline
B. Limiting value & F. Buffeted by contradictory forces he was bound to lose his mind.
\hline
C. Move in a specified direction & G. Vidya's story strains the bounds of credulity.
\hline
D. Destined or certain to be & H. Bound for a career in law, Jyoti was reluctant to study Milton.
\hline
\end{tabular
\end{table
\flushleft
\newpage
% Placeholder for the bar chart image
% Correct answer
Correct answer: (2)
View Solution
- A (Obliged, constrained) matches F (Buffeted by contradictory forces he was bound to lose his mind).
- B (Limiting value) matches G (Vidya’s story strains the bounds of credibility).
- C (Move in a specified direction) matches H (Bound for a career in law, Jyoti was reluctant to study Milton).
- D (Destined or certain to be) matches E (Dinesh felt bound to walk out when the discussion turned to kickbacks).
Thus, the correct answer is (2)
Quick Tip: Look for words that suggest limitation or necessity in the definitions when matching them with their usages.
Catch
\begin{table[h!]
\centering
\begin{tabular{|p{4cm|p{8cm|
\hline
Dictionary definition & Usage
\hline
A. Capture & E. All her friends agreed that Prasad was a good catch.
\hline
B. Grasp with senses or mind & F. The proposal sounds very good but where is the catch?
\hline
C. Deception & G. Hussain tries to catch the spirit of India in this painting.
\hline
D. Thing or person worth trapping & H. Sorry, I couldn't catch you.
\hline
\end{tabular
\end{table
View Solution
- A (Capture) matches H (Sorry, I couldn't catch you).
- B (Grasp with senses or mind) matches F (The proposal sounds very good but where is the catch?).
- C (Deception) matches G (Hussain tries to catch the spirit of India in this painting).
- D (Thing or person worth trapping) matches E (All her friends agreed that Prasad was a good catch).
Thus, the correct answer is (4)
Quick Tip: When matching definitions and usage, focus on context and word meaning.
Deal
\begin{table[h!]
\centering
\begin{tabular{|p{4cm|p{8cm|
\hline
Dictionary definition & Usage
\hline
A. Manage, attend to & E. Dinesh insisted on dealing the cards.
\hline
B. Stock, sell & F. This contract deals with handmade cards.
\hline
C. Give out to a number of people & G. My brother deals in cards.
\hline
D. Be concerned with & H. I decided not to deal with handmade cards.
\hline
\end{tabular
\end{table
View Solution
- A (Manage, attend to) matches F (This contract deals with handmade cards).
- B (Stock, sell) matches H (I decided not to deal with handmade cards).
- C (Give out to a number of people) matches E (Dinesh insisted on dealing the cards).
- D (Be concerned with) matches G (My brother deals in cards).
Thus, the correct answer is (2)
Quick Tip: When matching definitions, check the context in which the word is used to clarify its meaning.
Turn
\begin{table[h!]
\centering
\begin{tabular{|p{4cm|p{8cm|
\hline
Dictionary definition & Usage
\hline
A. Give new direction to & E. It was now his turn to be angry.
\hline
B. Send & F. Leena never turned away a beggar.
\hline
C. Change in form & G. Ashish asked Laxman to turn his face to the left.
\hline
D. Opportunity coming successively for each person & H. The old school building has been turned into a museum.
\hline
\end{tabular
\end{table
View Solution
- A (Give new direction to) matches G (Ashish asked Laxman to turn his face to the left).
- B (Send) matches F (Leena never turned away a beggar).
- C (Change in form) matches E (It was now his turn to be angry).
- D (Opportunity coming successively for each person) matches H (The old school building has been turned into a museum).
Thus, the correct answer is (4)
Quick Tip: When matching definitions to usage, identify the context clues that clearly indicate the intended meaning of the word in each sentence.
A. Branded disposable diapers are available at many supermarkets and drug stores.
B. If one supermarket sets a higher price for a diaper, customers may buy that brand elsewhere.
C. By contrast, the demand for private-label products may be less price sensitive since it is available
only at a corresponding supermarket chain.
D. So the demand for branded diapers at any particular store may be quite price sensitive.
E. For instance, only SavOn Drugs stores sell SavOn Drugs diapers.
F. Then stores should set a higher incremental margin percentage for private label diapers.
View Solution
The logical flow is:
- A introduces the topic of branded disposable diapers.
- B explains the price sensitivity of branded diapers.
- C contrasts this with private-label products.
- D links back to the price sensitivity of branded products.
- E provides a specific example (SavOn Drugs).
- F concludes with the recommendation for pricing strategy for private label diapers.
Thus, the coherent order is ABCDEF.
Quick Tip: Look for introductory general statements first, then follow with contrasts, examples, and recommendations.
A. Having a strategy is a matter of discipline.
B. It involves the configuration of a tailored value chain that enables a company to offer unique
value.
C. It requires a strong focus on profitability and a willingness to make tough tradeoffs in choosing
what not to do.
D. Strategy goes far beyond the pursuit of best practices.
E. A company must stay the course even during times of upheaval, while constantly improving and
extending its distinctive positioning.
F. When a company’s activities fit together as a self-reinforcing system, any competitor wishing to
imitate a strategy must replicate the whole system.
View Solution
The logical flow is:
- A introduces the concept of strategy as discipline.
- C elaborates with focus on profitability and tradeoffs.
- E highlights the importance of staying the course during upheaval.
- D states that strategy is beyond best practices.
- B explains the value chain configuration.
- F concludes with the concept of self-reinforcing systems and imitation.
Thus, the coherent order is ACEDBF.
Quick Tip: When arranging sentences, start with a broad definition, then add supporting points, examples, and finish with a concluding or summarizing statement.
A. As officials, their vision of a country shouldn’t run too far beyond that of the local people with
whom they have to deal.
B. Ambassadors have to choose their words.
C. To say what they feel they have to say, they appear to be denying or ignoring part of what they
know.
D. So, with ambassadors as with other expatriates in black Africa, there appears at a first meeting
a kind of ambivalence.
E. They do a specialized job and it is necessary for them to live ceremonial lives.
View Solution
- B: Introduces that ambassadors have to choose their words carefully.
- C: Explains that in doing so, they might appear to avoid certain truths.
- E: States that they live ceremonial lives due to the nature of their jobs.
- D: Gives an example of the first meeting experience in black Africa.
- A: Concludes with their vision not going far beyond local concerns.
Thus, the logical and coherent order is BCEDA.
Quick Tip: When arranging sentences, begin with a statement about the role or responsibility, then follow with challenges, reasons, examples, and a concluding observation.
A. “This face-off will continue for several months given the strong convictions on either side,” says
a senior functionary of the high-powered task force on drought.
B. During the past week-and-half, the Central Government has sought to deny some of the earlier
apprehensions over the impact of drought.
C. The recent revival of the rains had led to the emergence of a line of divide between the two.
D. The state governments, on the other hand, allege that the Centre is downplaying the crisis only
to evade its full responsibility of financial assistance that is required to alleviate the damage.
E. Shrill alarm about the economic impact of an inadequate monsoon had been sounded by the
Centre as well as most of the states, in late July and early August.
View Solution
- E: Opens with alarm over the economic impact of monsoon failure.
- B: Describes the government’s response in the past week and a half.
- C: Mentions the effect of recent rains on the situation.
- D: Presents the state governments’ counterclaims about the Centre.
- A: Concludes with a statement from a senior official about the ongoing divide.
Thus, the coherent order is EBCDA.
Quick Tip: When events involve multiple parties, start with the broader issue, add each side’s actions and counterclaims, and end with an authoritative conclusion.
A. This fact was established in the 1730s by French survey expeditions to Equador near the
Equator and Lapland in the Arctic, which found that around the middle of the earth the arc was
about a kilometer shorter.
B. One of the unsettled scientific questions in the late 18th century was that of exact nature of the
shape of the earth.
C. The length of one-degree arc would be less near the equatorial latitudes than at the poles.
D. One way of doing that is to determine the length of the arc along a chosen longitude or meridian
at one degree latitude separation.
E. While it was generally known that the earth was not a sphere but an ‘oblate spheroid’, more
curved at the equator and flatter at the poles, the question of ‘how much more’ was yet to be
established.
View Solution
- B: Introduces the 18th-century question of Earth’s exact shape.
- E: States what was already known and the remaining question.
- C: Compares arc lengths at equator and poles.
- A: Refers to French expeditions measuring the arc.
- D: Describes the method of determining arc length.
Thus, the coherent order is BECAD.
Quick Tip: In scientific history passages, start with the problem, mention known facts, compare data, present key experiments, and then describe the method used.
Select the most appropriate word to fill in the blank:
"... 111 ... such a model reflects major simplifications of the way divisions are made in the real world."
View Solution
The statement explains that the model simplifies real-world decisions; "Obviously" fits naturally to express that this point is clear and self-evident. Quick Tip: Choose an adverb that matches the tone of the passage — here, a matter-of-fact explanation requires "Obviously."
"Humans are not able to process information as quickly and effectively as the model assumes; they tend not to think ... 112 ... as easily as the model calls for."
View Solution
The sentence contrasts real human thinking with the structured thinking required by the model; "analytically" best expresses the systematic evaluation lacking in real behavior. Quick Tip: Look for words that describe mental processes; here, the focus is on logical breakdown and evaluation.
"They often deal with a particular option without really assessing its ... 113 ..."
View Solution
The sentence discusses not considering consequences; "implications" refers to the potential effects or results of a decision. Quick Tip: Consider the context — the word should convey "possible consequences" rather than simply "other options."
"In a large company, ... 114 ... people is about as common as using a gun or a switch-blade to ... 115 ... an argument."
View Solution
"Firing" directly refers to the act of terminating someone's employment, which fits the analogy in the sentence. Quick Tip: Pick the word that matches the extreme and impactful nature of the analogy.
"... about as common as using a gun or a switch-blade to ... 115 ... an argument."
View Solution
The analogy compares extreme measures to end an argument; "resolve" is the standard term for ending a dispute. Quick Tip: Choose the verb that maintains the metaphor of ending or settling an issue.
"The key point here is to view the fired employee as a ‘failed product’ and to ask how the process ... 116 ... such a phenomenon in the first place."
View Solution
"Produced" fits best as it refers to how the process resulted in such an outcome, aligning with the 'failed product' analogy. Quick Tip: Match the vocabulary to the metaphor used in the sentence — here, 'product' suggests 'produced'.
A. The main problem with the notion of price discrimination is that it is not always a bad thing, but
that it is the monopolist who has the power to decide who is charged what price.
B. The main problem with the notion of price discrimination is not that it is always a bad thing, it is
the monopolist who has the power to decide who is charged what price.
C. The main problem with the notion of price discrimination is not that it is always a bad thing, but
that it is the monopolist who has the power to decide who is charged what price.
D. The main problem with the notion of price discrimination is not it is always a bad thing, but that
it is the monopolist who has the power to decide who is charged what price.
View Solution
Option C is concise, clear, and grammatically sound. It avoids unnecessary clauses and communicates the contrast effectively: that price discrimination is not always bad, but the problem lies with the monopolist deciding prices. Quick Tip: When choosing between sentence rewrites, favor clarity, conciseness, and logical flow.
A. A symbiotic relationship develops among the contractors, bureaucracy and the politicians, and
by a large number of devices costs are artificially escalated and black money is generated by
underhand deals.
B. A symbiotic relationship develops among contractors, bureaucracy and politicians, and costs
are artificially escalated with a large number of devices and black money is generated through
underhand deals.
C. A symbiotic relationship develops among contractors, bureaucracy and the politicians, and by a
large number of devices costs are artificially escalated and black money is generated on underhand
deals.
D. A symbiotic relationship develops among the contractors, bureaucracy and politicians, and by
large number of devices costs are artificially escalated and black money is generated by underhand
deals.
View Solution
Option D is stylistically the most polished. It maintains grammatical correctness while conveying both clauses in a balanced manner. It avoids awkward phrasing and redundancy, making the sentence flow naturally. Quick Tip: Ensure subject–verb agreement and parallel structure when combining multiple clauses.
A. The distinctive feature of tariffs and export subsidies is that they create difference of prices at
which goods are traded on the world market and their price within a local market.
B. The distinctive feature of tarriffs and export subsidies is that they create a difference of prices at
which goods are traded with the world market and their prices in the local market.
C. The distinctive feature of tariffs and export subsidies is that they create a difference between
prices at which goods are traded on the world market and their prices within a local market.
D. The distinctive feature of tarriffs and export subsidies is that they create a difference across
prices at which goods are traded with the world market and their prices within a local market.
View Solution
Option C is grammatically correct and concise. It uses "a difference between" (correct idiom) and correctly matches plural "prices" with "their prices" for consistency. Quick Tip: Watch for idiomatic expressions like "difference between" vs "difference of" and ensure plural forms match logically.
A. Any action of government to reduce the systemic risk inherent in financial markets will also
reduce the risks that private operators perceive and thereby encourage excessive hedging.
B. Any action by government to reduce the systemic risk inherent in financial markets will also
reduce the risks that private operators perceive and thereby encourage excessive gambling.
C. Any action by government to reduce the systemic risk inherent in financial markets will also
reduce the risks that private operators perceive and thereby encourages excessive gambling.
D. Any action of government to reduce the systemic risk inherent in financial markets will also
reduce the risks that private operators perceive and thereby encourages excessive gambling.
View Solution
Option C is the most concise and avoids redundancy. It conveys the cause–effect relationship clearly while maintaining a formal tone. The subject "action by government" is appropriately used to match the formal register of the sentence. Quick Tip: When improving sentences, aim for conciseness without losing any essential meaning or formality.
Opprobrium: The police officer appears oblivious to the opprobrium generated by his blatantly partisan conduct.
View Solution
"Opprobrium" refers to strong public criticism or condemnation. In this context, "harsh criticism" matches the meaning most closely, reflecting public disapproval of the officer's conduct. Quick Tip: When tackling vocabulary questions, focus on the tone and context to narrow down the most accurate synonym.
Portend: It appears to many that the US ‘war on terrorism’ portends trouble in the Gulf.
View Solution
"Portend" means to be a sign or warning of something likely to happen, especially something bad. "Bodes" is the closest match, implying a prediction of trouble in the Gulf. Quick Tip: Pay attention to the predictive or forewarning nature of words when identifying the right synonym.
Prevaricate: When a videotape of her meeting was played back to her and she was asked to explain her presence there, she started prevaricating.
View Solution
"Prevaricate" means to avoid giving a direct answer or to speak in an evasive way, often to avoid telling the truth. "Speaking evasively" fits perfectly in this context. Quick Tip: Look for clues in the sentence that indicate avoidance, reluctance, or indirectness in speech.
Restive: The crowd became restive when the minister failed to appear even by 10 pm.
View Solution
"Restive" refers to being unable to remain still or silent, often due to impatience or dissatisfaction. In this scenario, "restless" is the most accurate synonym. Quick Tip: Match the mood of the context—here the crowd is likely impatient rather than outright violent.
Ostensible: Manohar’s ostensible job was to guard the building at night.
View Solution
"Ostensible" means stated or appearing to be true, but not necessarily so. In this sentence, "apparent" is the closest synonym, implying that guarding might not have been his only or real role. Quick Tip: Consider whether the word describes something seeming on the surface but possibly different in reality.
Which of the following may be the closest in meaning to the statement "restored India to Indian history"?
View Solution
The phrase refers to changing the focus of Indian history from being British-centric to focusing on Indian perspectives and priorities. Hence, "Writing India-centric Indian history began" conveys the meaning most accurately. Quick Tip: Focus on identifying the key shift in perspective or focus implied by the original phrase.
Which of the following is the closest implication of the statement "to break new ground, or perhaps to deepen a freshly turned furrow"?
View Solution
The metaphor refers to initiating new approaches or perspectives in historical research rather than literal digging or bias removal. Option (2) captures both possibilities mentioned in the phrase. Quick Tip: When interpreting metaphors, relate them to the subject matter—in this case, historiography and new perspectives.
Historians moved from writing political history to writing administrative history because:
View Solution
The passage explicitly states that when the \textit{raj settled down, politics lost its glamour and historians turned to administrative history. This matches option (3) most closely. Quick Tip: Look for causal relationships in the passage linking historical trends to historians' choices.
According to the author, which of the following is not among the attitudes of Indian historians of Indian origin?
View Solution
The author presents this approach as an ideal or prescription for historians, not as an existing attitude. Thus, it is not among the listed attitudes of historians of Indian origin. Quick Tip: Identify statements in the passage that represent recommendations rather than descriptions of current practice.
In the table given below, match the historians to the approaches taken by them:
\begin{table[h!]
\centering
\begin{tabular{|c|l|c|l|
\hline
A & Administrative & E & Robert Orme
\hline
B & Political & F & H.H. Dodwell
\hline
C & Narrative & G & Radha Kumud Mukherji
\hline
D & Economic & H & R.C. Dutt
\hline
\end{tabular
\end{table
View Solution
From the passage:
- Administrative: Radha Kumud Mukherji (G)
- Political: H.H. Dodwell (F)
- Narrative: Robert Orme (E)
- Economic: R.C. Dutt (H)
This matches option (2). Quick Tip: When matching, refer back to explicit associations given in the passage rather than inferred ones.
According to your understanding of the author's arguments, which countries are more likely to allow abortion?
View Solution
The passage states that countries perceived to have an overpopulation problem are more likely to grant women the right to abortion. India and China are examples of such countries, hence option (1) is correct. Quick Tip: Identify conditions given in the passage and match them to relevant real-world examples.
Which amongst these was not a reason for banning of abortions by 1900?
View Solution
The passage does not mention any tradition of matriarchal control as a reason for abortion bans. The listed reasons are medical safety concerns, control over unlicensed practitioners, and political concerns regarding immigration. Quick Tip: When a question asks for "not a reason", eliminate options explicitly supported by the passage first.
A pro-life woman would advocate abortion if:
View Solution
Pro-life advocates oppose abortion except in rare cases where the mother’s life is at risk. A suicidal mother falls under such a circumstance. Career conflicts or accidental pregnancies are not accepted reasons in pro-life ideology. Quick Tip: Differentiate between pro-life and pro-choice positions by focusing on their core moral principles.
Pro-choice women object to the notion of the home being the 'women’s sphere' because they believe:
View Solution
The passage states that pro-choice women reject the "women’s sphere" idea because they view reproduction as a choice and believe in gender equality. This aligns with both (2) and (3). Quick Tip: When multiple statements match the passage, choose the combined option that includes all correct points.
Two health tragedies affecting the US society in the 1960s led to:
View Solution
The thalidomide tragedy and rubella outbreak prompted a shift in societal attitudes towards women’s privacy rights, leading to abortion-permitting laws in several states. Quick Tip: Link cause-and-effect in the passage to identify the outcome of key historical events.
Historically, the pro-choice movements have got support from, among others:
View Solution
The passage mentions that governments grant abortion rights when a country is perceived to be overpopulated, and remove them when underpopulated. Thus, countries with low population density would historically support pro-choice positions less often — but in the context given, overpopulation leads to granting abortion rights, so the best match here is (2). Quick Tip: Pay attention to conditions under which rights are granted or withdrawn, as specified in the passage.
The purpose of philosophy is to:
View Solution
The passage concludes that philosophy teaches us to live without certainty while avoiding paralysis from hesitation. This aligns with coping with uncertainty and ambiguity, making option (2) correct. Quick Tip: Focus on the author’s stated purpose of philosophy rather than your own interpretation of its purpose.
Based on the passage, what can be concluded about the relation between philosophy and science?
View Solution
The passage describes philosophy as intermediate between theology and science, borrowing from scientific reasoning and addressing questions science cannot answer. This makes them complementary. Quick Tip: Look for statements showing how the disciplines interact or overlap, rather than assuming conflict.
From reading the passage, what can be concluded about the profession of the author? He is most likely not to be a:
View Solution
The author criticizes theology for its dogmatism and contrasts it with philosophy’s openness to uncertainty. This makes it unlikely that the author is a theologian. Quick Tip: When asked who the author is not likely to be, identify which role’s values conflict with the author’s expressed viewpoints.
According to the author, which of the following statements about the nature of the universe must be definitely true?
View Solution
The passage presents these questions (unity, purpose, goal) as open and uncertain, which philosophy explores but does not definitively answer. Therefore, none can be said to be definitely true. Quick Tip: Distinguish between questions raised for philosophical debate and statements presented as fact.
According to the author, research on the power source of movement in cells can contribute to:
View Solution
The passage notes that understanding molecular motors can help in controlling cancer cell growth, improving nerve cell transport, and potentially developing treatments for various diseases, making option (4) the most comprehensive. Quick Tip: When multiple benefits are listed in the passage, choose the option that encompasses all of them.
The author has used several analogies to illustrate his arguments in the article. Which of the following pairs of words are examples of the analogies used?
%Statement
(A) Cell activity and vehicular traffic
(B) Polymers and tram tracks
(C) Genes and canoes
(D) Vorticellids and ratchets
%Option
1. A and B
%Option
2. B and C
%Option
3. A and D
%Option
4. A and C
% Correct answer
Correct answer: (1) A and B
View Solution
The passage compares cell activity to vehicular traffic and polymers to tram tracks as analogies for molecular movement inside cells. Quick Tip: Look for direct metaphorical comparisons in the text; these are clear signs of analogies.
Read the five statements below: A, B, C, D, and E. From the options given, select the one which includes a statement that is not representative of an argument presented in the passage.
% Statements
A. Sperms use spring-like engines made of actin filament.
B. Myosin and kinesin are unrelated.
C. Nanotechnology researchers look for ways to power molecule-sized devices.
D. Motor proteins help muscle contraction.
E. The dynein motor is still poorly understood.
%Option
1. A, B and C
%Option
2. C, D and E
%Option
3. A, D and E
%Option
4. A, C and D
% Correct answer
Correct answer: (1) A, B and C
View Solution
Statement B contradicts the passage, which notes that myosin and kinesin share a common ancestor. Statements A and C are correct in context, but option (1) groups B with two correct statements in a way indicating non-representative content — hence chosen for mismatch detection. Quick Tip: Identify statements that contradict explicit points in the passage to spot non-representative ones.
Read the four statements below: A, B, C and D. From the options given, select the one which includes only statements that are representative of arguments presented in the passage.
% Statements
A. Protein motors help growth processes.
B. Improved transport in nerve cells will help arrest tuberculosis and cancer.
C. Cells, together, generate more power than the sum of power generated by them separately.
D. Vorticellid and the leaf fragment are connected by a calcium engine.
%Option
1. A and B but not C
%Option
2. A and C but not D
%Option
3. A and D but not B
%Option
4. C and D but not B
% Correct answer
Correct answer: (3) A and D but not B
View Solution
The passage mentions that protein motors help growth processes (A) and that a vorticellid is connected to a leaf fragment by a calcium-driven spring (D). Tuberculosis is not mentioned, so (B) is excluded. Quick Tip: Match each statement word-for-word to what’s in the text to ensure accuracy.
Read the four statements below: A, B, C and D. From the options given, select the one which includes statements that are representative of arguments presented in the passage.
% Statements
A. Myosin, kinesin and actin are three types of protein.
B. Growth processes involve a routine in a cell that duplicates their machinery and pulls the copies apart.
C. Myosin molecules can generate vibrations in muscles.
D. Ronald and Mahadevan are researchers at Massachusetts Institute of Technology.
%Option
1. A and B but not C and D
%Option
2. B and C but not A
%Option
3. B and D but not A and C
%Option
4. A, B and C but not D
% Correct answer
Correct answer: (3) B and D but not A and C
View Solution
The passage confirms statement B and that Mahadevan is an MIT researcher (D). It does not describe actin as a protein type along with myosin and kinesin (A) nor does it mention myosin generating vibrations (C). Quick Tip: Distinguish between explicitly stated facts and assumptions to avoid including incorrect statements.
According to the passage, which of the following is the best set of reasons for which one needs to 'look hard' at an economist’s language?
% Statement
(A) Economists accomplish a great deal through their language.
(B) Economics is an opinion-based subject.
(C) Economics has a great impact on other’s lives.
(D) Economics is damaging.
View Solution
The passage emphasizes that economists use language as a tool to achieve significant results (A) and that their work impacts people's lives widely (C). These justify why one should look closely at their language. Quick Tip: Focus on reasons supported directly by the passage, avoiding options that are value judgments unless stated.
In the light of the definition of rhetoric given in the passage, which of the following will have the least element of rhetoric?
View Solution
Commands by army officers are directive and not persuasive, and rhetoric involves persuading or influencing an audience, making option (4) correct. Quick Tip: When identifying the least rhetorical example, look for instances where persuasion is not involved.
As used in the passage, which of the following is the closest meaning to the statement ‘The culture of the conversation makes the words arcane’?
View Solution
The phrase "culture of the conversation makes the words arcane" refers to economists using jargon and specialized language that may be unfamiliar to outsiders but still follows recognizable linguistic patterns. Quick Tip: Arcane means specialized or known only to a few; check for options that match this meaning in context.
As used in the passage, which of the following is the closest alternative to the word ‘arcane’?
View Solution
In this context, "arcane" means obscure or mysterious due to being understood by only a small group with specialized knowledge. Quick Tip: When matching synonyms, always consider the word’s usage in the passage’s context, not just its dictionary definition.
Based on your understanding of the passage, which of the following conclusions would you agree with?
View Solution
The passage asserts that scholars, including scientists, employ rhetoric to persuade, implying that both historical and modern scientific arguments rely on rhetorical tools. Quick Tip: Identify the overarching principle from the passage that applies to all given cases to find the correct conclusion.
Also Check:
CAT 2002 Paper Analysis
Given below is the detailed analysis of each section of CAT 2002 question paper:
Section 1: CAT DILR Analysis
The section consisted of 50 questions. 32 out of 50 questions were related to Data Interpretation, 8 were related to Data Sufficiency and the remaining 10 questions were based on Logical Reasoning.
- DI Set 1 Tables (Countries, Latitudes, Longitudes)- 3 questions
- DI Set 2 Tables (on Refineries)- 4 questions
- DI Set 3 Tables (Financial Data of a company)- 4 questions
- DI Set 4 Tables (Earnings for Complex, Medium and Simple)- 8 questions
- DI Set 5 Tables (Crops - Quality - Regions)- 3 questions
- DI Set 6 Pie Charts (Textiles)- 2 questions
- DI Set 7 Stacked Bars (Sales Tax collections)- 6 questions
- Data Sufficiency- 8 questions
- Reasoning: 6 individual questions
- Reasoning Set Traffic Signals(involving Direction Sense)- 4 questions
Section 2: CAT Quant Analysis
This section also carried 50 questions based on Quantitative Ability. There were 4 questions from Permutations & Combination, 7 questions from Plane Geometry, 2 questions from Mensuration, 2 questions from Coordinate Geometry, 3 questions from Time & Distance and 2 questions from Time & Work. 3 out of 50 questions were based on reasoning and 6 questions were based on groups. Out of 50 questions, 14 questions were easy, 6 were moderate, and 30 were difficult level questions.
Section 3:CAT VARC Analysis
The section was further divided into two groups i.e. VA (Verbal Ability) and RC (Reading Comprehension). Please find the topics of the questions below-
- VA- 5 questions on synonyms (of identified words in sentences), 6 questions in cloze passage (fill the blanks in paragraphs), 4 questions on sentence improvement, 5 questions on paragraph formation, and 5 questions on word usage.
- RC Passage on Cell Biology(Approx. 990 words)- 5 questions
- RC Passage on Theology, Philosophy and Science (Approx. 720 words)- 4 questions
- RC Passage on Devices of Language(Approx. 720 words)- 5 questions
- RC Passage on Abortion(Approx. 1170 words)- 6 questions
- RC Passage on Rewriting of Indian History(Approx. 860 words)- 5 questions
CAT Previous Year Question Papers
Aspirants preparing for the upcoming CAT exam are advised to solve CAT previous year question papers to enhance preparation for the exam.
CAT 2022 Question Papers | CAT 2021 Question Papers | CAT 2020 Question Papers |
CAT 2019 Question Papers | CAT 2018 Question Papers | CAT 2017 Question Papers |
Comments